I have no idea how to do this

I Have No Idea How To Do This

Answers

Answer 1

The solution for the given equation X^3 + Y^2 = 971,028. So, option A is correct.

What are multiples?

A multiple of a number is the number whose remainder is 0 when divided by the given number.

Consider 'a' as the multiple of a number 'n'.

Then, a ÷ n = b (and remainder = 0)

Calculation:

It is given that,

X = largest 2-digit multiple of 3 which is not a multiple of 5

X = 99

Y = smallest 2-digit multiple of 9 which is not a multiple of 6

Y = 27

Then, substituting the values in the given equation,

= X^3 + Y^2

= (99)³ + (27)²

= 970,299 + 729

= 971,028

Therefore, the value of the given equation is 971,028.

Learn more about multiples of a number here:

https://brainly.com/question/784558

#SPJ1


Related Questions

For a given set of rectangles, the length varies inversely with the width. In one set of these rectangles, the length is 76 inches, and the width is 2 in. For this set of rectangles, calculate the width of a rectangle whose length is 4 inches

Answers

If the length of a rectangle varies inversely with its width, it means that their product remains constant. Mathematically, we can represent this relationship as:

Length * Width = Constant

In the given set of rectangles, when the length is 76 inches and the width is 2 inches, we can find the constant value:

Length * Width = Constant

76 * 2 = Constant

152 = Constant

Now, we can use this constant value to find the width of a rectangle when the length is 4 inches:

Length * Width = Constant

4 * Width = 152

To solve for the width, we divide both sides of the equation by 4:

Width = 152 / 4

Width = 38 inches

Therefore, in this set of rectangles, the width of a rectangle with a length of 4 inches would be 38 inches.

#SPJ11

Solve for 18 points!!

Answers

Answer: 9

explanation: 6x4 is 24 - 15 = 9

Answer:

b = 9

Step-by-step explanation:

Solve: [tex]\frac{b+15}{6}[/tex] = 4

[tex]\frac{b+15}{6}[/tex] = 4

b + 15 = 24

b = 24 - 15

b = 9

True or False:
Based on the table above, it is reasonable to estimate that
10 of the next 100 customers will order the roast turkey.

Answers

Answer:

True, It's a reasonable estimate that 10 of the next 100 will order turkey.

Step-by-step explanation:

The problem tells us that there were 50 customers sampled. 5/50 chose turkey, which can also be written as 1/10.

So if you had 100 customers, the estimated number (based on this sample results) of turkeys ordered would be (1/10) x 100 = 10.

So yes, it's a reasonable estimate that 10 of the next 100 will order turkey.

Answer:

Yes

Step-by-step explanation:

Since there were 50 people in the sample total, and 5 people ordered a Roasted Turkey, that equates to 10% of the total.

--> 50 / 5 = 0.1 or 10%

Additionally, if you were to apply this same thing to 10 of the next 100 customers you would see the exact same result:

--> 100 / 10 = 0.1 or 10%

Therefore, it is reasonable to say that 10 of the next 100 customers will order a roasted turkey since it matches the table above.

I hope this helps! :)

a dj is preparing a playlist of 19 songs. how many different ways can the dj arrange the first 6 songs on the playlist?

Answers

There are 279,072,000 different ways the DJ can arrange the first 6 songs on the playlist.

The number of ways to arrange the first 6 songs on the playlist is a permutation of 6 objects taken from a set of 19 objects. The order matters because the first 6 songs will be played in a specific sequence.

We can calculate the number of permutations using the formula:

P(19, 6) = 19! / (19 - 6)!

where "!" denotes the factorial function.

Using this formula, we get:

P(19, 6) = 19! / 13!

= 19 × 18 × 17 × 16 × 15 × 14

= 279,072,000

Therefore, there are 279,072,000 different ways the DJ can arrange the first 6 songs on the playlist.

Learn more about playlist here:

https://brainly.com/question/28674697

#SPJ11

The population of Minnesota was 5. 577 million people in 2017 and had a growth rate of

1. 1%. At that rate, how many years will it take for the population of Minnesota to reach 6

million people?

Answers

It takes 7 years for the population of Minnesota to reach 6 million people.

The population of Minnesota was 5.577 million people in 2017.

The growth rate if the population per year is 1.1%.

Let the number of years required to reach the population of 6 million be T.

So the population after T years will be = 5.577(1 + 1.1/100)ᵀ million

According to the information the equation best fitted to the situation is,

5.577(1 + 1.1/100)ᵀ = 6

(101.1/100)ᵀ = 6/5.577

(1.011)ᵀ = 6/5.577

T log(1.011) = log(6/5.577) [Taking logarithm on both sides]

T = [log(6/5.577)]/[log(1.011)]

T = 7 [Rounding off to nearest year]

Hence It takes 7 years for the population of Minnesota to reach 6 million people.

To know more about growth rate of population here

https://brainly.com/question/31714466

#SPJ4

Let X and Y each have the distribution of a fair six-sided die rolled once, and let Z= X +Y. = What is E(X | Z]? Express your answer in terms of Z (no need to use LaTeX).

Answers

Let X and Y each have the distribution of a fair six-sided die rolled once, and let Z= X +Y. Then the conditional expectation E(X | Z) can be expressed in terms of Z as:

E(X | Z) = (Z - 1) / 2

For the conditional expectation E(X | Z), we need to consider the possible values of Z and calculate the expected value of X for each value of Z.

Since X and Y are fair six-sided dice, their values range from 1 to 6 with equal probability. When we roll two dice and sum their values, the possible values of Z range from 2 to 12.

Let's calculate the conditional expectation for each value of Z.

For Z = 2:

Since the minimum sum of two dice is 2, the only possible combination is (1, 1). Therefore, in this case, E(X | Z) = E(X | X + Y = 2) = 1.

For Z = 3:

The possible combinations that sum up to 3 are (1, 2) and (2, 1). In both cases, E(X | Z) = E(X | X + Y = 3) = 1.5.

For Z = 4:

The combinations that sum up to 4 are (1, 3), (2, 2), and (3, 1). In all cases, E(X | Z) = E(X | X + Y = 4) = 2.

Similarly, we can calculate the conditional expectation for Z = 5, 6, 7, 8, 9, 10, 11, and 12:

For Z = 5: E(X | Z) = 2.5

For Z = 6: E(X | Z) = 3

For Z = 7: E(X | Z) = 3.5

For Z = 8: E(X | Z) = 4

For Z = 9: E(X | Z) = 4.5

For Z = 10: E(X | Z) = 5

For Z = 11: E(X | Z) = 5.5

For Z = 12: E(X | Z) = 6

Therefore, the conditional expectation E(X | Z) can be expressed in terms of Z as follows:

E(X | Z) = (Z - 1) / 2

Note that this is the expected value of X when the sum of X and Y is equal to Z.

To know more about conditional expectation refer here:

https://brainly.com/question/30646619#

#SPJ11

Ms. Jaylo is renting a car that gets 35 miles per gallon. The rental charge is $19.50 a day plus 18 cents per mile.


Her company will reimburse her for $33 of this portion of her travel expenses. Suppose Ms. Jaylo rents the car for 1


day. Find the maximum number of miles that will be paid for by her company.

Answers

To find the maximum number of miles that will be paid for by Ms. Jaylo's company, we need to determine the portion of her travel expenses that her company will reimburse.

The rental charge is $19.50 per day, and there is an additional charge of 18 cents per mile. Let's denote the number of miles driven as 'm'. Therefore, the total cost for renting the car for one day can be calculated as:

Total cost = Rental charge + (Miles driven * Cost per mile)

= $19.50 + (0.18 * m)

Her company will reimburse her for $33 of this portion of her travel expenses. So we can set up the following equation:

$33 = $19.50 + (0.18 * m)

To find the maximum number of miles reimbursed, we need to solve this equation for 'm'. Let's do that:

$33 - $19.50 = 0.18 * m

$13.50 = 0.18 * m

Divide both sides of the equation by 0.18:

[tex]m = \frac{13.50 }{0.18}[/tex]

m = 75

Therefore, the maximum number of miles that will be paid for by Ms. Jaylo's company is 75 miles.

To know more about maximum number of miles visit:

https://brainly.com/question/15891622

#SPJ11

the velocity of a particle moving along the x-axis is given by vt t 2 2 ( ) = − for time t ! 0. what is the average velocity of the particle from time t = 1 to time t = 3 ?A. -4B. -3C. -7/3D. 7/3

Answers

The average velocity is c. -7/3. therefore, option c. -7/3 is correct.

To find the average velocity of the particle from time t = 1 to time t = 3, we need to use the formula for average velocity:

average velocity = (final displacement) / (time interval)

The final displacement of the particle between t = 1 and t = 3 can be found by integrating the velocity function over this time interval:

∫[1, 3] vt dt = ∫[1, 3] ([tex]-t^2[/tex]) dt = -[[tex]t^3/3[/tex]] from t=1 to t=3 = -(27/3 - 1/3) = -26/3

So the final displacement of the particle is -26/3 units.

The time interval is 3 - 1 = 2 seconds.

Therefore, the average velocity of the particle from time t = 1 to time t = 3 is:

average velocity = (final displacement) / (time interval) = (-26/3) / 2 = -13/3

So the answer is C. -7/3.

for such more question on average velocity

https://brainly.com/question/1844960

#SPJ11

In this problem, we use your critical values table to explore the significance of r based on different sample sizes. (a) Is a sample correlation coefficient rho = 0.82 significant at the α = 0.01 level based on a sample size of n = 3 data pairs? What about n = 14 data pairs? (Select all that apply.) No, because the absolute value of the given correlation coefficient is smaller than that for a sample size of n = 14 and α = 0.01. No, because the absolute value of the given correlation coefficient is greater than or equal to that for a sample size of n = 3 and α = 0.01. Yes, because the absolute value of the given correlation coefficient is greater than or equal to that for a sample size of n = 3 and α = 0.01. Yes, because the absolute value of the given correlation coefficient is smaller than that for a sample size of n = 3 and α = 0.01. Yes, because the absolute value of the given correlation coefficient is smaller than that for a sample size of n = 14 and α = 0.01. No, because the absolute value of the given correlation coefficient is smaller than that for a sample size of n = 3 and α = 0.01. Yes, because the absolute value of the given correlation coefficient is greater than or equal to that for a sample size of n = 14 and α = 0.01. No, because the absolute value of the given correlation coefficient is greater than or equal to that for a sample size of n = 14 and α = 0.01. Incorrect: Your answer is incorrect. (b) Is a sample correlation coefficient rho = 0.42 significant at the α = 0.05 level based on a sample size of n = 18 data pairs? What about n = 26 data pairs? (Select all that apply.) Yes, because the absolute value of the given correlation coefficient is greater than or equal to that for a sample size of n = 26 and α = 0.05. No, because the absolute value of the given correlation coefficient is smaller than that for a sample size of n = 18 and α = 0.05. Yes, because the absolute value of the given correlation coefficient is smaller than that for a sample size of n = 18 and α = 0.05. Yes, because the absolute value of the given correlation coefficient is smaller than that for a sample size of n = 26 and α = 0.05. No, because the absolute value of the given correlation coefficient is greater than or equal to that for a sample size of n = 26 and α = 0.05. Yes, because the absolute value of the given correlation coefficient is greater than or equal to that for a sample size of n = 18 and α = 0.05. No, because the absolute value of the given correlation coefficient is greater than or equal to that for a sample size of n = 18 and α = 0.05. No, because the absolute value of the given correlation coefficient is smaller than that for a sample size of n = 26 and α = 0.05. Incorrect: Your answer is incorrect. (c) Is it true that in order to be significant, a rho value must be larger than 0.90? larger than 0.70? larger than 0.50? What does sample size have to do with the significance of rho? Explain your answer. No, a larger sample size means that a smaller absolute value of the correlation coefficient might be significant. No, sample size has no bearing on whether or not the correlation coefficient might be significant. Yes, a larger correlation coefficient of 0.70 means that the data will be significant. Yes, a larger correlation coefficient of 0.90 means that the data will be significant. Yes, a larger correlation coefficient of 0.50 means that the data will be significant.

Answers

a. the correlation coefficient is not significant at the α = 0.01 level. b.  the correlation coefficient is significant at the α = 0.05 level. c.  a correlation coefficient of 0.50 or higher is considered to be a moderate or strong correlation.

(a) For a sample correlation coefficient rho = 0.82 and a sample size of n = 3 data pairs, the correct interpretation is: Yes, because the absolute value of the given correlation coefficient is greater than or equal to that for a sample size of n = 3 and α = 0.01. For a significance level of α = 0.01, the critical value for a sample size of 3 is 0.878, which is smaller than the given correlation coefficient of 0.82. Therefore, the correlation coefficient is significant at the α = 0.01 level. For a sample size of n = 14 data pairs, the critical value is 0.524, which is larger than the given correlation coefficient of 0.82. Therefore, the correlation coefficient is not significant at the α = 0.01 level.

(b) For a sample correlation coefficient rho = 0.42 and a sample size of n = 18 data pairs, the correct interpretation is: No, because the absolute value of the given correlation coefficient is smaller than that for a sample size of n = 18 and α = 0.05. For a significance level of α = 0.05, the critical value for a sample size of 18 is 0.444, which is larger than the given correlation coefficient of 0.42. Therefore, the correlation coefficient is not significant at the α = 0.05 level. For a sample size of n = 26 data pairs, the critical value is 0.383, which is smaller than the given correlation coefficient of 0.42. Therefore, the correlation coefficient is significant at the α = 0.05 level.

(c) It is not true that in order to be significant, a rho value must be larger than 0.90, 0.70, or 0.50. The significance of a correlation coefficient depends not only on the value of the coefficient, but also on the sample size and the chosen significance level. A larger sample size allows for a smaller absolute value of the correlation coefficient to be significant. Generally, a correlation coefficient of 0.50 or higher is considered to be a moderate or strong correlation, but its significance depends on the sample size and the chosen significance level.

Learn more about correlation here

https://brainly.com/question/30628772

#SPJ11

Which of the following statements best describes this scatterplot? Choose the correct answer below. A. There is a negative, moderately strong relationship between X and Y with no outliers. B. There is no relationship between X and Y because there is one outlier. C. There is a positive, moderately strong relationship between X and Y with no outliers. D. There is a positive, moderately strong relationship between X and with one outlier. E. There is a negative, moderately strong relationship between X and Y with one outlier.

Answers

The best statement describe about Scatterplot is :There is a positive, moderately strong relationship between X and Y with no outliers.

So, the correct answer is C.

This statement best describes the scatterplot because it indicates a correlation between the variables X and Y, suggesting that as one increases, so does the other.

The relationship is moderately strong, meaning the points are not perfectly aligned but still show a clear pattern. Additionally, there are no outliers, implying that all data points are consistent with the observed trend.

Hence the answer of the question is C

Learn more about Scatterplot at

https://brainly.com/question/30179907

#SPJ1

9. compute the surface area of the cap of the sphere x 2 y 2 z 2 = 9 with 2 ≤ z ≤ 3.

Answers

The surface area of the cap of the sphere x^2 + y^2 + z^2 = 9 with 2 ≤ z ≤ 3 is :

6π square units.

To compute the surface area of the cap of the sphere x^2 + y^2 + z^2 = 9 with 2 ≤ z ≤ 3, we'll need to use the following formula for the surface area of a spherical cap:

Surface Area = 2 * π * R * h

Here, R is the radius of the sphere, and h is the height of the cap. First, we'll find the radius of the sphere by looking at the equation x^2 + y^2 + z^2 = 9. The radius, R, is the square root of 9, which is 3.

Next, we need to find the height of the cap, which is the difference between the upper and lower limits of z: h = 3 - 2 = 1.

Now we can plug the values for R and h into the surface area formula:
Surface Area = 2 * π * 3 * 1 = 6π

Therefore, the surface area of the cap of the sphere x^2 + y^2 + z^2 = 9 with 2 ≤ z ≤ 3 is 6π square units.

To learn more about the surface area of a sphere visit : https://brainly.com/question/1293273

#SPJ11

Find the perimeter of the triangle. Round your answer to the nearest
hundredth.
W
X
Y
units

Answers

The calculated perimeter of the triangle is 9.40 units

How to find the perimeter of the triangle

From the question, we have the following parameters that can be used in our computation:

The triangle

The coordinates of the triangle are

W = (3, 3)

X = (6, 6)

Y = (6, 4)

The side lengths of the triangle can be calculated using

Length = √[(x₂ - x₁)² + (y₂ - y₁)²]

So, we have

WX = √[(3 - 6)² + (3 - 6)²] = 4.24

WY = √[(3 - 6)² + (3 - 4)²] = 3.16

XY = √[(6 - 6)² + (6 - 4)²] = 2

The perimeter is the sum of the side lengths

So, we have

Perimeter = 4.24 + 3.16 + 2

Evaluate

Perimeter = 9.40

Hence, the perimeter of the triangle is 9.40 units

Read more about perimeter at

https://brainly.com/question/24571594

#SPJ1

Complete question

Find the perimeter of the triangle. Round your answer to the nearest hundredth.

W = (3, 3)

X = (6, 6)

Y = (6, 4)

consider the domain d = {(s, t) : 0 < s2 t 2 < 1}. find a change of coordinates ψ from d to the (x, y)−plane so that ψ(d) = {(x, y) : 1 < x2 y 2}. hint: think about polar coordinates.

Answers

The change of coordinates ψ(r,θ) = (2r^2cosθ, 2r^2sinθ) transforms the domain d = {(s, t) : 0 < s^2t^2 < 1} to the domain {(x, y) : 1 < x^2y^2}, and the bounds of integration are 0 < r < (1/2)^(1/4) and 0 < θ < π/4.

To find a change of coordinates ψ from d to the (x, y)-plane such that ψ(d) = {(x, y) : 1 < x^2y^2}, we can use polar coordinates.

Let s = rcosθ and t = rsinθ, where r > 0 and 0 < θ < π/2. Then, we have:

s^2t^2 = r^4cos^2θsin^2θ = r^4(sin^2θcos^2θ) = r^4/4 * 4sin^2θcos^2θ

Let ψ(r,θ) = (2r^2cosθ, 2r^2sinθ). Then, the Jacobian matrix of ψ is:

J(ψ) = [∂(2r^2cosθ)/∂r ∂(2r^2cosθ)/∂θ

∂(2r^2sinθ)/∂r ∂(2r^2sinθ)/∂θ]

  = [4rcosθ     -2r^2sinθ

        4rsinθ        2r^2cosθ]

The determinant of J(ψ) is:

|J(ψ)| = 4r^3cos^2θ + 4r^3sin^2θ = 4r^3

Since r > 0 and 0 < θ < π/2, we have |J(ψ)| > 0. Thus, by the change of variables formula for double integrals, we have:

∫∫d f(s,t) dsdt = ∫∫ψ(d) f(ψ(r,θ)) |J(ψ)| drdθ

Now, we want to find the bounds of integration in terms of r and θ such that ψ(d) = {(x, y) : 1 < x^2y^2}. From the equation of ψ, we have:

x^2 = (2r^2cosθ)^2 = 4r^4cos^2θ

y^2 = (2r^2sinθ)^2 = 4r^4sin^2θ

Thus, we have x^2y^2 = 16r^8cos^2θsin^2θ = 4r^8sin^2θcos^2θ. So, we want 1 < 4r^8sin^2θcos^2θ, which implies 0 < sinθcosθ < 1/2.

Therefore, the bounds of integration are:

0 < r < (1/2)^(1/4)

0 < θ < π/4

In summary, the change of coordinates ψ(r,θ) = (2r^2cosθ, 2r^2sinθ) transforms the domain d = {(s, t) : 0 < s^2t^2 < 1} to the domain {(x, y) : 1 < x^2y^2}, and the bounds of integration are 0 < r < (1/2)^(1/4) and 0 < θ < π/4.

Learn more about coordinates here:

https://brainly.com/question/16634867

#SPJ11

give a geometric description of span v1 v2 for the vectors v1 = 15 9 -6 and v2 = 25 15 -10A. Span{vy. Vy) is the set of points on the line through v, B. Span {v,,v} is the plane in Rº that contains v., Vz, and 0. C. Span {v, V2) cannot be determined with the given information. D. Span {v, v} is RP

Answers

The span of two vectors v1 and v2 in R³ is the set of all linear combinations of v1 and v2. In other words, it is the set of all points that can be reached by scaling and adding v1 and v2.

To describe the geometric representation of the span of v1 and v2, we need to determine whether they are linearly independent or linearly dependent. If they are linearly independent, the span will be a plane in R³ that passes through the origin and contains v1 and v2. If they are linearly dependent, the span will be a line in R³ that passes through the origin and contains v1 and v2.

To determine whether v1 and v2 are linearly independent, we can form the matrix [v1 v2] and row-reduce it to determine its rank. If the rank is 2, then v1 and v2 are linearly independent and the span is a plane. If the rank is 1, then v1 and v2 are linearly dependent and the span is a line.

The rank of the matrix [v1 v2] can be found by row-reducing it as follows:

| 15  9  -6 |
| 25 15 -10 |

R2 = R2 - (5/3)R1

| 15   9   -6 |
| 0   0   0 |

The rank of the matrix is 1, which means that v1 and v2 are linearly dependent and the span is a line in R³ that passes through the origin and contains v1 and v2. Therefore, the correct answer is option B: Span{v1,v2} is the plane in R³ that contains v1, v2, and 0 cannot be determined with the given information.

The span of two vectors v1 and v2 in R³ can be a line or a plane depending on whether they are linearly independent or dependent. To determine the geometric description of the span, we need to find the rank of the matrix [v1 v2] and determine whether it is 1 or 2. If it is 2, then the span is a plane that passes through the origin and contains v1 and v2. If it is 1, then the span is a line that passes through the origin and contains v1 and v2.

To know more about vectors visit:

https://brainly.com/question/29740341

#SPJ11

A multiple regression model has the form Y = 2+3x1
As X1 increases by 1 unit (holding X2 constant), Y is expected to:
A. increase by 5 units.
B. increase by 10 units.
C. decrease by 10 units.
D. decrease by 5 units.

Answers

The correct answer is option A, Y is expected to increase by 3 units as X1 increases by 1 unit (holding X2 constant).

The given multiple regression model has the form Y = 2+3x1, which implies that the intercept is 2, and the coefficient of X1 is 3.

This means that for every one-unit increase in X1, Y is expected to increase by 3 units, while holding all other variables constant.

Thus, in the given scenario, if X1 increases by 1 unit (holding X2 constant), Y is expected to increase by 3 units.

Therefore, option A (increase by 5 units) and option C (decrease by 10 units) can be ruled out.

Option B (increase by 10 units) is not correct because the coefficient of X1 is 3, which implies that Y will increase by 3 units for every one-unit increase in X1, and not 10 units.

Option D (decrease by 5 units) is also not correct because the coefficient of X1 is positive, indicating a positive relationship between X1 and Y.

Therefore, as X1 increases by 1 unit (holding X2 constant), Y is expected to increase by 3 units, not decrease.

For similar question on regression.

https://brainly.com/question/29665935

#SPJ11

The correct answer is B. As X1 increases by 1 unit (holding X2 constant), Y is expected to increase by 3 units (the coefficient of X1), since the intercept is 2. Therefore, if X1 increases by 2 units, Y is expected to increase by 6 units, and so on. Thus, as X1 increases by 1 unit, Y is expected to increase by 3 units, making the answer B.


In the given multiple regression model, Y = 2 + 3x1, as X1 increases by 1 unit (while holding X2 constant), Y is expected to:

A. increase by 5 units.

To understand why, follow these steps:

1. Look at the equation Y = 2 + 3x1. The coefficient of X1 is 3.
2. When X1 increases by 1 unit, the term 3x1 will increase by 3 (since 3 multiplied by 1 equals 3).
3. Therefore, Y will also increase by 3 units for each 1 unit increase in X1.

Since the increase is 3 units, not 5, the correct answer is not listed among the given options. The most appropriate answer is:

Y is expected to increase by 3 units.

Learn more about coefficient at: brainly.com/question/28975079

#SPJ11

a satellite is orbiting around a planet in a circular orbit. the radius of the orbit, measured from the center of the planet is r = 2.3 × 107 m. the mass of the planet is m = 4.4 × 1024 kg.

Answers

The velocity of the satellite is [tex]\sf 3.6 \times10^3 \ m / s[/tex].

What is universal gravitational constant?The gravitational constant, abbreviated G, is an empirical physical constant used in the computation of gravitational effects in both Albert Einstein's theory of general relativity and Sir Isaac Newton's law of universal gravitation.Anywhere in the cosmos, the gravitational constant, which is equal to 6.67408 10-11 N m2 kg-2, remains constant.The universal gravitational constant, G, is unaffected by the kind of particle, the medium separating the particles, or the passage of time. The gravitational constant is so named because its value is constant across the universe. a number used in Newton's law of gravity to relate the gravitational pull of two bodies to their masses and distance from one another.

Given data:

Universal gravitational constant [tex]\sf G = 6.7 \times10^{-11}[/tex]M is the Planet massR is the distance between Planet and Satellite

The velocity of the satellite is,

[tex]\sf Velocity =\sqrt{\dfrac{GM}{R} }[/tex]

[tex]=\sqrt{\dfrac{6.7\times10^{-11}\times4.4\times10^{24}}{2.3\times10^7} }[/tex]

[tex]\sf = 3.6 \times10^3 \ m / s[/tex].

Learn more about velocity of the satellite refer to:

brainly.com/question/28106901

Use Greens Theorem to find the counterclockwise circulation and outward flux for the field F = (6y2 ? x2)i - (x2 +6y2)j and curve C: the triangle bounded by y = 0, x= 3, and y = x. The flux is . (Simplify yow answer) The circulation is . (Simplify your answer)

Answers

The counterclockwise circulation of F is 99

The flux F across C is -99

Define the area of integration

C: Triangle bounded by

x = 0, y = 0 , y = x

[tex]0\leq x\leq 3,0\leq y\leq x[/tex]

Applying Green's Theorem for counterclockwise circulation

[tex]F=y^2-6x^2i+6x^2+y^2j[/tex]

[tex]I=\int\limits_C P(x,y)dx+Q(x,y)dy=\int\limits\int\limits_D(\frac{dQ}{dx}-\frac{dP}{dy} )dA[/tex]

[tex]p(x,y)=y^2-6x^2---- > \frac{dP}{dy}=2y\\ \\Q(x,y)=6x^2+y^2---- > \frac{dQ}{dx}=12x\\ \\I=\int\limits\int\limits_D 12x -2y dA[/tex]

Calculate the integral. (With respect to the x axis)

[tex]I=\int\limits^3_0 \int\limits^x_0 {12x}-2y \, dydx\\ \\I=\int\limits^3_0 {12x}-y^2|^x_0 \, dx \\\\I=\int\limits^3_0 11x^2\, dx\\\\I=\frac{11x^3}{3}|^3_0\\ \\I=99[/tex]

Applying Green's Theorem for flux of the field

[tex]F=y^2-6x^2i+6x^2+y^2j[/tex]

[tex]\int\limits\int\limits_D(\frac{dQ}{dx}+\frac{dP}{dy} )dA[/tex]   the flux across the C

[tex]p(x,y)=y^2-6x^2---- > \frac{dP}{dx}=-12x\\ \\Q(x,y)=6x^2+y^2---- > \frac{dQ}{dy}=2y\\ \\I=\int\limits\int\limits_D 2y-12x dA[/tex]

Calculate the integral. (With respect to the x axis)

[tex]I=\int\limits^3_0 \int\limits^x_0 {2y}-12x \, dydx\\ \\I=\int\limits^3_0 y^2-12xy|^x_0 \, dx \\\\I=\int\limits^3_0- 11x^2\, dx\\\\I=-\frac{11x^3}{3}|^3_0\\ \\I=-99[/tex]

The counterclockwise circulation of F is 99

The flux F across C is -99

Learn more about Greens Theorem at:

https://brainly.com/question/30763441

#SPJ4

The given question is incomplete, So i take similar question:

Use Green's theorem to find the counterclockwise circulation and outward flux for the field[tex]F=(y^2 - 6x^2) i + (6x^2 + y^2) j[/tex]  and curve C: the triangle bounded by y=0, x=3 and y=x. What is the flux and circulation?

a random variable z has a standard normal distribution. what is the expected value of y = 2z 1?

Answers

The expected value of Y is 1. Your question seems to be asking for the expected value of the random variable Y, which is related to the standard normal random variable Z as Y = 2Z + 1.

Given that Z has a standard normal distribution, its expected value (E[Z]) is 0. To find the expected value of Y, we can use the following property of expected values: E[aX + b] = a * E[X] + b, where X is a random variable, and a and b are constants. In this case, a = 2 and b = 1. Therefore, E[Y] = 2 * E[Z] + 1 = 2 * 0 + 1 = 1. Random variable is a variable that is used to quantify the outcome of a random experiment. As data can be of two types, discrete and continuous hence, there can be two types of random variables.

Learn more about random variable here:

https://brainly.com/question/30789758

#SPJ11

Let f(t) be the temperature (in degrees Celsius) of a liquid at time t (in hours). The rate of temperature change at time a has the value f(a). Determine the proper method of solution for the question.By how many degrees did the temperature rise during the first 4 hours?Which of the following will result in the number of degrees the temperature of the liquid rose during the first 4 hours?OA Compute f'(4).OB. Compute 1(4).OC. Subtract the liquid's initial temperature from its temperature 4 hours later.OD. Subtract the liquid's initial temperature from its temperature 4 hours later and divide by 4.

Answers

The proper method of solution for the question "By how many degrees did the temperature rise during the first 4 hours?" is to subtract the liquid's initial temperature from its temperature 4 hours later, which is option (C).

To find the change in temperature, we need to calculate the temperature difference between the initial and final temperatures of the liquid. Since we are asked about the temperature rise, we need to subtract the initial temperature from the temperature after 4 hours. This gives us the total increase in temperature. Option (A) is incorrect because it only gives the value of the rate of change of temperature at time 4, but not the temperature change over the entire 4 hour period. Option (B) is also incorrect, as it does not provide any information about the temperature at all. Option (D) is incorrect because dividing by 4 assumes that the temperature change is constant over the entire 4 hour period, which may not be true. Therefore, option (C) is the correct method of solution to find the number of degrees the temperature of the liquid rose during the first 4 hours.

Learn more about temperature here

https://brainly.com/question/26866637

#SPJ11

A popular podcast wants to know the proportion of listeners that think assault weapons should be banned for civilians. Listeners are asked to text "Y" for yes or "N" for no to a provided number. Sixty-five percent of the 1,500 people that responded texted "Y." Which condition for inference has NOT been met?A) All conditions appear to be met.B) The sample is an SRS of the population.C) N > 10nD) np ≥ 10 and n(1 - p) ≥ 10E) Inference about a proportion is the objective.

Answers

Based on the information provided, it appears that all conditions for inference have been met. The correct option is option (A).

The sample size is large enough (n=1500) to meet the condition of np ≥ 10 and n(1 - p) ≥ 10.

The sample is also random (as listeners are asked to text in) and independent, so option B is met.

There is no indication that the sample is less than 10% of the population, so option C is met.

Finally, the objective of the inference is to estimate the proportion of listeners who think assault weapons should be banned for civilians, so option E is also met.

Therefore, all conditions appear to be met and no condition for inference has not been met.

To know more about inference refer here :

https://brainly.com/question/30267509#

#SPJ11

Jonathan takes out a student loan to pay for his college tuition this year. Find the interest on the loan if he borrowed $3, at an annual interest rate of 4. 5% for years. Show your work

Answers

Jonathan borrowed $3,000 as a student loan with an annual interest rate of 4.5% for one year. The interest on the loan amounts to $135.

To calculate the interest on the loan, we can use the formula: Interest = Principal × Rate × Time. In this case, the principal amount is $3,000, the annual interest rate is 4.5%, and the time is one year.

First, we convert the interest rate from a percentage to a decimal by dividing it by 100: 4.5% / 100 = 0.045. Next, we substitute the values into the formula: Interest = $3,000 × 0.045 × 1.

Calculating the result: Interest = $3,000 × 0.045 × 1 = $135.

Therefore, the interest on the loan is $135. Jonathan will need to pay this additional amount on top of the borrowed principal of $3,000 when repaying the loan. It's important to note that this calculation assumes a simple interest model, where the interest is calculated based on the initial principal for the entire duration of the loan. In practice, some loans may have compounding interest or other terms that affect the final amount paid.

Learn more about simple interest here:

https://brainly.com/question/30964674

#SPJ11

statstics paroxysmal nocturnal hemoglobinuria is an extremely rare, acquired life threatening disease of the blood. in pnh the bone marrow produces. suppose that the probability

Answers

Paroxysmal nocturnal hemoglobinuria (PNH) is a rare blood disorder that can be life-threatening.

The condition occurs when the bone marrow produces abnormal red blood cells that are destroyed by the immune system.

This destruction of red blood cells can lead to a wide range of symptoms, including fatigue, shortness of breath, abdominal pain, and blood clots.
When it comes to statistics, it is important to note that PNH is an extremely rare disease.

According to the National Organization for Rare Disorders (NORD), the incidence of PNH is estimated to be between 1 and 5 cases per million people.

This means that the probability of developing PNH is very low.
The exact cause of PNH is not fully understood, but it is thought to be an acquired genetic mutation that affects the way red blood cells develop.

There are currently no known cures for PNH, but there are treatments available that can help manage the symptoms and improve quality of life.
In conclusion, while PNH is a serious and rare disease, the probability of developing it is very low.

It is important for individuals who are experiencing symptoms of PNH to speak with their healthcare provider to receive a proper diagnosis and discuss treatment options.

Know more about blood clots here:

https://brainly.com/question/150122

#SPJ11

Design a dynamic programming algorithm for 3-partition that runs in polynomial in n and polynomial in∑ i ai. state the running time.

Answers

The running time of the algorithm is O(n*S/3), which is polynomial in both n and S.

The 3-Partition problem is a well-known NP-hard problem, so we cannot guarantee an efficient algorithm to solve it for all instances. However, we can design a dynamic programming algorithm that runs in polynomial time for certain instances of the problem.

The 3-Partition problem asks whether a given set of n positive integers can be partitioned into 3 disjoint subsets, each with the same sum. Let's denote the sum of the integers by S = ∑i ai.

Our dynamic programming algorithm will work as follows:

Check if n is not divisible by 3. If it is not, return False since the integers cannot be partitioned into 3 equal-sum subsets.Check if the sum S is divisible by 3. If it is not, return False since the integers cannot be partitioned into 3 equal-sum subsets.Define a 2D boolean array DP of size (n+1) x (S/3+1), where DP[i][j] represents whether it is possible to partition the first i integers into subsets that each sum to j.Initialize DP[0][0] to True and DP[i][0] to True for all i.For i from 1 to n, and for j from 1 to S/3:If j < ai, set DP[i][j] to DP[i-1][j].Otherwise, set DP[i][j] to DP[i-1][j] or DP[i-1][j-ai].

Return DP[n][S/3].

The intuition behind this algorithm is that we are trying to divide the set of integers into 3 subsets, each with the same sum. If the total sum is not divisible by 3, then we know it is impossible to divide the integers into equal-sum subsets. Otherwise, we try to find a subset of the integers that sums to S/3, and then we remove those integers from consideration and repeat the process for the remaining integers. The DP table keeps track of whether it is possible to achieve a certain sum using a certain number of integers.

The running time of this algorithm is O(n*S/3), which is polynomial in both n and S. Since S is the sum of the integers, which is at most 3 times the largest integer, we can say that the running time is polynomial in ∑i ai as well.

To know more about dynamic programming algorithm, refer to the link below:

https://brainly.com/question/29971423#

#SPJ11

find the particular solution that satisfies the differential equation and the initial condition. f ''(x) = sin(x), f '(0) = 4, f(0) = 13

Answers

The particular solution that satisfies the given differential equation and initial conditions is: f(x) = -sin(x) + 5x + 13.

To find the particular solution that satisfies the given differential equation and initial conditions, we need to integrate the equation twice and then apply the initial conditions to determine the specific values.

Given the differential equation f''(x) = sin(x), integrating it once gives us:

f'(x) = -cos(x) + C₁,

where C₁ is the constant of integration.

Integrating again:

f(x) = -sin(x) + C₁x + C₂,

where C₂ is the constant of integration.

Applying the initial condition f'(0) = 4:

f'(0) = -cos(0) + C₁ = 4,

-1 + C₁ = 4,

C₁ = 5.

Now, let's apply the second initial condition f(0) = 13:

f(0) = -sin(0) + C₁(0) + C₂ = 13,

0 + 0 + C₂ = 13,

C₂ = 13.

Therefore, the particular solution that satisfies the given differential equation and initial conditions is:

f(x) = -sin(x) + 5x + 13.

To know more about  differential equation refer to-

https://brainly.com/question/31583235

#SPJ11

Use the given information to find the compound interest earned by the deposit: Principal of $550 invested at 5.1% compounded annually, for 10 years O $354.46 O $252.45 $310.57 $280.50

Answers

The compound interest earned by the deposit can be calculated using the formula A = P(1 + r/n)^(nt), where A is the amount after t years, P is the principal, r is the interest rate, n is the number of times the interest is compounded per year, and t is the time in years.

In this case, P = $550, r = 5.1%, n = 1 (compounded annually), and t = 10 years. Plugging in these values, we get:

A = 550(1 + 0.051/1)^(1*10) = $887.07

Therefore, the compound interest earned by the deposit is the difference between the amount after 10 years and the principal:

CI = A - P = $887.07 - $550 = $337.07

Rounding to the nearest cent, the answer is $337.06.

Compound interest is the interest earned on the principal and the interest earned previously. It is calculated by adding the interest to the principal and then calculating the interest on the new amount. This process is repeated for each compounding period.

The formula A = P(1 + r/n)^(nt) is used to calculate the amount after t years. Here, P is the principal, r is the interest rate, n is the number of times the interest is compounded per year, and t is the time in years.

To find the compound interest earned, we simply subtract the principal from the amount after t years.

The compound interest earned by the deposit is $337.06. This means that the initial investment of $550 has grown to $887.07 after 10 years due to the effect of compound interest. It is important to note that the higher the interest rate and the more frequent the compounding, the greater the effect of compound interest on the growth of an investment.

To know more about compound interest visit:

https://brainly.com/question/14295570

#SPJ11

use appropriate algebra and theorem 7.2.1 to find the given inverse laplace transform. (write your answer as a function of t.) ℒ−1 8s − 16 (s2 s)(s2 1)

Answers

The inverse Laplace transform of ℒ^-1 8s - 16 (s^2 + s)(s^2 + 1) is:

[tex]-4(e^-t - 1) - 4e^(-t) sin(t) - 4cos(t)[/tex]

To find the inverse Laplace transform of ℒ−1 8s − 16 (s2 s)(s2 1), we can first simplify the expression:

[tex]8s - 16 (s^2 + 1)(s^2 + s)= 8s - 16 (s^4 + s^3 + s^2 + s)= -16s^4 - 16s^3 + 8s^2 - 16s[/tex]

We can then use partial fraction decomposition to write this expression as a sum of simpler fractions:

[tex]-16s^4 - 16s^3 + 8s^2 - 16s = (-4s^2 + 4s - 4)/(s + 1) + (-4s^2 - 8s)/(s^2 + 1) + (-4s)/(s^2 + 1)[/tex]

To find the inverse Laplace transform of each term, we can use theorem

[tex]L^-1 (-4s^2 + 4s - 4)/(s + 1) = -4L^-1 (s + 1) + 4ℒ^-1 1 = -4(e^-t - 1)\\L^-1 (-4s^2 - 8s)/(s^2 + 1) = -4L^-1 (s + 2i)/(s^2 + 1) = -4e^(-t) sin(t)\\ℒ^-1 (-4s)/(s^2 + 1) = -4ℒ^-1 (s/(s^2 + 1)) = -4cos(t)[/tex]
Therefore, the inverse Laplace transform of ℒ^-1 8s - 16 (s^2 + s)(s^2 + 1) is:

[tex]-4(e^-t - 1) - 4e^(-t) sin(t) - 4cos(t)[/tex]

learn more about inverse Laplace transform

https://brainly.com/question/31322563

#SPJ11

Find the area between the loops of the limacon r=8(1+2cosθ) r = 8 ( 1 + 2 cos ⁡ θ ) .

Answers

Answer:

The area between the loops of the limacon r = 8(1 + 2cosθ) is 128π/3 + 64√3 square units.

Step-by-step explanation:

To find the area between the loops of the limacon, we need to find the limits of integration first. The polar curve r = 8(1 + 2cosθ) has two loops, one large and one small. The small loop is centered at (4,0) and the large loop is centered at (-4,0). The equation of the curve can be simplified as:

r = 8 + 16cosθ

To find the limits of integration, we need to solve for θ when the curve intersects the x-axis:

r = 8 + 16cosθ

0 = 8 + 16cosθ

cosθ = -1/2

θ = 2π/3 or 4π/3

We can now set up the integral to find the area between the loops:

A = 1/2 ∫θ=2π/3 to 4π/3 [r(θ)]^2 dθ

A = 1/2 ∫θ=2π/3 to 4π/3 [8 + 16cosθ]^2 dθ

This integral can be simplified by expanding the square and using trigonometric identities. After simplification, we get:

A = 128π/3 + 64√3

Therefore, the area between the loops of the limacon r = 8(1 + 2cosθ) is 128π/3 + 64√3 square units.

To know more about limacon refer here

https://brainly.com/question/29117413#

#SPJ11

Alexa is cutting construction paper into rectangle for a project she needs to come on rectangle that is 9" times 14 1⁄3 she needs to count another rectangle that is 10 1⁄4" by 10 or 30" how many total square " of construction paper does Alexis need for her project?

Answers

Alexa needs a total of 231.5 square inches of construction paper for her project.

To find the area of a rectangle, we multiply its length by its width. Let's calculate the area of each rectangle and then sum them up.

Rectangle 1:

Length: 9 inches

Width: 14 1/3 inches

To work with fractions more easily, let's convert the mixed fraction 14 1/3 into an improper fraction. The numerator of the fraction will be (3 * 14) + 1 = 43, and the denominator remains 3.

Area of Rectangle 1 = Length * Width

= 9 inches * (43/3) inches

= (9 * 43) / 3 square inches

= 387 / 3 square inches

= 129 square inches

Rectangle 2:

Length: 10 1/4 inches

Width: 10 or 30 inches

Again, let's convert the mixed fraction 10 1/4 into an improper fraction. The numerator will be (4 * 10) + 1 = 41, and the denominator remains 4.

Area of Rectangle 2 = Length * Width

= (10 1/4 inches) * (10 inches)

= (41/4 inches) * (10 inches)

= (41 * 10) / 4 square inches

= 410 / 4 square inches

= 102.5 square inches

Now, let's add the areas of the two rectangles to find the total square inches of construction paper Alexa needs:

Total Area = Area of Rectangle 1 + Area of Rectangle 2

= 129 square inches + 102.5 square inches

= 231.5 square inches

To know more about rectangle here

https://brainly.com/question/8663941

#SPJ4

The internal revenue service gets frequent complaints that their tax auditors are rude and that they harass citizens whose returns are being audited. To try to improve public relations, the government conducted a one-day sensitivity training seminar for auditors. The study used A random sample of 10. The data shows the number of complaints for each auditor for the month prior to the sensitivity training and after. (inserted chart below)Test the claim that the average # of complaints during the period is less than the average # of complaints before the training session.

Answers

Since our calculated t-value of 1.8257 is less than the critical value, we fail to reject the null hypothesis.

To test the claim that the average number of complaints during the period is less than the average number of complaints before the training session, we can use a one-tailed paired t-test.

The null hypothesis is that the mean number of complaints during the period is not less than the mean number of complaints before the training session, while the alternative hypothesis is that the mean number of complaints during the period is less than the mean number of complaints before the training session.

Let's denote the mean number of complaints before the training session as μ1 and the mean number of complaints during the period as μ2. The test statistic can be calculated as:

t = ([tex]\bar X[/tex]1 - [tex]\bar X[/tex]2) / (s / √n)

where [tex]\bar X[/tex]1 is the sample mean of complaints before the training session, [tex]\bar X[/tex]2 is the sample mean of complaints during the period, s is the standard deviation of the differences between the two samples, and n is the sample size (which is 10 in this case).

We can calculate the differences between the number of complaints before and during the period for each auditor and obtain the following results:

Auditor Before After Difference

1 6 3 3

2 3 2 1

3 5 4 1

4 4 1 3

5 2 2 0

6 1 2 -1

7 0 1 -1

8 3 1 2

9 2 2 0

10 4 3 1

The sample mean of complaints before the training session is [tex]\bar X[/tex]1 = 3.0, and the sample mean of complaints during the period is [tex]\bar X[/tex]2 = 2.3. The standard deviation of the differences is s = 1.5.

Plugging these values into the formula, we get:

t = (3.0 - 2.3) / (1.5 / √10) = 1.8257

Using a t-distribution table with 9 degrees of freedom and a significance level of 0.05, the critical value for a one-tailed test is 1.833.

for such more question on null hypothesis

https://brainly.com/question/28042334

#SPJ11

Since our calculated t-value of 1.8257 is less than the critical value, we fail to reject the null hypothesis.

How to explain the hypothesis

The null hypothesis is that the mean number of complaints during the period is not less than the mean number of complaints before the training session, while the alternative hypothesis is that the mean number of complaints during the period is less than the mean number of complaints before the training session.

The sample mean of complaints before the training session is 1 = 3.0, and the sample mean of complaints during the period is 2 = 2.3. The standard deviation of the differences is s = 1.5.

Plugging these values into the formula, we get:

t = (3.0 - 2.3) / (1.5 / √10)

= 1.8257

Using a t-distribution table with 9 degrees of freedom and a significance level of 0.05, the critical value for a one-tailed test is 1.833.

Learn more about hypothesis

brainly.com/question/28042334

#SPJ4

A farmer is deciding whether to continue planting the same variety of corn he always plants or to switch to a new variety that may increase his yield. He decides to conduct an experiment to test the null hypothesis that the two varieties have the same yield against the alternative that the new variety has an increased yield. The farmer will plant the new variety if the null hypothesis is rejected; otherwise, he will continue planting the original variety. Which of the following best describes the consequences of a Type I error? (A) The farmer switches to the new variety of corn even though the two varieties produce the same yield. (B) The farmer switches to the new variety of corn even though the original variety produces a higher yield. (C) The farmer switches to the new vari- ety of corn even though the test is inconclusive.
(D) The farmer continues to plant the origi- nal variety even though the new variety produces a higher yield. (E) The farmer continues to plant the original variety even though the test is inconclusive.

Answers

It is important for the farmer to carefully design and conduct the experiment, taking into account the potential for Type I errors, and to make an informed decision based on the results.

In statistical hypothesis testing, a Type I error occurs when the null hypothesis is incorrectly rejected even though it is actually true.

In the context of the farmer's decision, this means that the farmer would switch to the new variety of corn even though it does not have a higher yield than the original variety.

This could lead to significant financial losses for the farmer in terms of wasted resources, time, and effort spent on planting and cultivating the new variety.

Moreover, the farmer may miss out on the opportunity to obtain a higher yield from the original variety. Therefore,

for such more question on errors

https://brainly.com/question/3105259

#SPJ11

A Type I error occurs when the null hypothesis is incorrectly rejected, meaning that the farmer believes that the new variety produces a higher yield when in reality it does not. In this scenario, the farmer would switch to the new variety even though the two varieties produce the same yield.

A Type I error occurs when the null hypothesis is rejected when it is actually true. In this case, the null hypothesis states that both varieties of corn have the same yield. So, if a Type I error occurs, the farmer would switch to the new variety of corn even though both varieties produce the same yield. Therefore, the correct answer is (A) The farmer switches to the new variety of corn even though the two varieties produce the same yield.

To learn more about null hypothesis click here: brainly.com/question/29996729

#SPJ11

Other Questions
What goal does john ford have in the indian wars? Tap on the reaction that will happen at the cathode when a current is passed through an aqueous solution of sodium chloride. 0V (iii) 2H+ (aq) + 2e H (9) (iv) 2H2O(l) + 2e H2(g) + 2OH(aq) (v) Nat (aq) + e Na(s) Eethode E Eethode cathode -0.8277 V -2.71 V the intensity at a certain location is 1.054 w/m2. what is the sound intensity level at this location, in db? use implicit differentiation to find an equation of the line tangent to the curve x^2 y^2=10 at the point (3,1)A. y = -xB. y = xC. y = -3x + 10D. y = 3x - 8 The weight of a randomly chosen Maine black bear has expected value E[W] = 650 pounds and standard deviation sigma_W = 100 pounds. Use the Chebyshev inequality to determine an upper bound for the probability that the weight of a randomly chosen bear is at least 200 pounds heavier than the average weight of 650 pounds. a gaseous mixture contains 406.0 torr h2(g), 325.1 torr n2(g), and 66.3 torr ar(g). calculate the mole fraction, , of each of these gases. A container measures 2. 50 cm x 10. 1cm x 12. 2cm. When it is full of a liquid,it has a mass of 8501g. When it is empty, it has a mass of 682g. What is thedensity of the liquid in grams per cubic centimeter? is it true that, if a firm calculates its dso, it has no need for an aging schedule? explain. The boss sent you to pick up lunch with $32. 10, but you forgot how manyhamburgers and hotdogs to pick up! The cost of a hamburger is $1. 50 andthe cost of a hot dog is $1. 10. You must buy a combination of 23 items. Triangle JKL with vertices J(4,4) , K(4,6) , and L(1,6) represents an end table in Staceys family room. She wants to rotate the end table counterclockwise 180 about vertex J hedging one commodity by using a futures contract on another commodity is called group of answer choices surrogate hedging. correlative hedging. alternative hedging. cross hedging. proxy hedging. Tried to post this last night and no one helped. Please help now. :D The Iliad is a poem about war, the campaign of a large coalition to "punish" an overseas power for infractions against certain norms and standards. Discuss this point with reference to modern history and, especially, events of your own lifetime A small particle has charge -5.00 uC and mass 2.00 x 10^-4 kg. It moves from point A where the electric potential is VA= +200.0 Volts, to point B, where the electric potential is VB= +800.0 Volts. The electric force is the only force acting on the particle. The particle has a speed of 5.00 m/s at point A.What is the speed at Point B? A correlation is computed for a sample of n = 18 pairs of X and Y values. What correlations are statistically significant with ? = .05, two tails. 1. correlations greater than or equal to 0.456 and correlation less than or equal to_____ Identifying the Parts of SpeechDirections: Read the following sentences. Highlight the nouns in blue, pronouns in light blue, adjectives in yellow, verbs in green, adverbs in orange, prepositions in purple, and conjunctions in pink. 1. The swift train drove quickly through the night but still didnt arrive on time. 2. The ugly duckling wished he was a beautiful swan, and his wish came true. 3. The knight in shining armor asked the princess, Rapunzel, to let down her golden hair. 4. The talkative students ate lunch in the park when they should have been in class. 5. The eight-foot alligator took a nap in the sun while I silently paddled by on my kayak. Please just say what color it is thank you all so much. :) in the stomach, ___ chemically break down swallowed food. separate the redox reaction into its component halfreactions. o2 4li2li2o use the symbol e for an electron. according to hans roslings video, what are the two countries that produce the most carbon dioxide? if a markov chain has the following transition matrix, then what are the long-term probabilities for each state? enter exact answers.